You are on page 1of 10

In order to answer this question, we need to know if the slopes of the reflection and the slope of the second

line are equal. When we reflect a line around the line y = x, the x and y coordinates of each point on the original line will switch. In other words, if the point (a,b) lies on the original line, then the point (b, a) will lie on its reflection. The slope of the reflection will be the reciprocal of the slope of the original line. This means that if p = 1/m, then the answer to our question will be yes. (1) As we said, if p * m = 1, then the reflection of the line with slope p will be parallel to m. Using the equation given us in (1), we substitute in for m: p * (p + 2) = 1 p2 + 2p - 1 = 0 Using the quadratic formula, we can see that if lines will be parallel. INSUFFICIENT (2) We can use the same approach as with (1). p * (3p) = 1 3p2 = 1 , then the

We still cannot reach a definitive conclusion. INSUFFICIENT Combining the statements, we see that there is not a shared solution for p. The lines will not be parallel. SUFFICIENT The credited response is (C) BOTH statements TOGETHER are sufficient, but NEITHER statement ALONE is sufficient.

A line with the equation y = px + q is reflected over the line y = x. Is the reflection of this line parallel to the line y = mx + n? (1) m = p + 2 (2) m = 3p

Choices

A Statement (1) ALONE is sufficient, but statement (2) alone is not sufficient. B Statement (2) ALONE is sufficient, but statement (1) alone is not sufficient. Your answer was incorrect C BOTH statements TOGETHER are sufficient, but NEITHER statement ALONE is sufficient. D EACH statement ALONE is sufficient. E Statements (1) and (2) TOGETHER are NOT sufficient to answer the question asked, and additional data are needed.
In order to answer this question, we need to know if the slopes of the reflection and the slope of the second line are equal. When we reflect a line around the line y = x, the x and y coordinates of each point on the original line will switch. In other words, if the point (a,b) lies on the original line, then the point (b, a) will lie on its reflection. The slope of the reflection will be the reciprocal of the slope of the original line. This means that if p = 1/m, then the answer to our question will be yes. (1) As we said, if p * m = 1, then the reflection of the line with slope p will be parallel to m.

Using the equation given us in (1), we substitute in for m: p * (p + 2) = 1 p2 + 2p - 1 = 0 Using the quadratic formula, we can see that if lines will be parallel. INSUFFICIENT (2) We can use the same approach as with (1). p * (3p) = 1 3p2 = 1 , then the

We still cannot reach a definitive conclusion. INSUFFICIENT Combining the statements, we see that there is not a shared solution for p. The lines will not be parallel. SUFFICIENT The credited response is (C) BOTH statements TOGETHER are sufficient, but NEITHER statement ALONE is sufficient. A line with the equation y = px + q is reflected over the line y = x. Is the reflection of this line parallel to the line y = mx + n? (1) m = p + 2 (2) m = 3p

Choices

Statement (1) ALONE is sufficient, but statement (2) alone is not sufficient. B Statement (2) ALONE is sufficient, but statement (1) alone is not sufficient. C BOTH statements TOGETHER are sufficient, but NEITHER statement ALONE is sufficient. Your answer was correct! D EACH statement ALONE is sufficient. E Statements (1) and (2) TOGETHER are NOT sufficient to answer the question asked, and additional data are needed.
While you wait for others to answer

What types of activities would you like to do while you wait for other players to answer?
Please check any that apply to you: Math Flash Cards Play a word game. Learn test taking strategies Rate your confidence on the question you just answered Help your teammate(s) with the question you just answered other (please share as many ideas as you like)

Submit

Quick Survey
Question Information

Difficulty: 32pts Skills: Parallel Lines Quadratic Equations

Algebra Geometry Lines Data Sufficiency

Grockit Messages

Correct! You gained 16 XP.

Correct! You gained 4 XP.

Incorrect. You lost 4 XP.

Incorrect. You lost 4 XP.

Correct! You gained 4 XP.

Correct! You gained 32 XP.

You just reached level 3! Share on Facebook

Correct! You gained 8 XP.

Correct! You gained 16 XP.

Incorrect. You lost 8 XP.

Correct! You gained 2 XP.

Correct! You gained 16 XP.

Correct! You gained 16 XP.

Correct! You gained 32 XP.

Hint Skip
Are you sure you want a hint? You will only get 1/2 the XP if you answer correctly.

As an example, consider 325.

We are given that f(c) = 16f(k). Let c be the integer lmn, and k be the integer pqr.

That is, since 16 is a power of 2 (and not divisible by 3 or 5), the only difference between the 2 numbers must be in the first digit, the hundreds digit. Since the tens' and ones' digits are the same, when we subtract, they will cancel out. (e.g. 353 - 153 = 200). l = p+4, so c must be four hundred greater than k. Algebraically: 100l + 10m + k - (100p + 10q + r) = ? (We determined that m = q, and k =

r, so those cancel.) 100l - 100p = ? Substitute l = p + 4 100(p + 4) - 100p = 100p + 400 - 100p = 400. The function g is defined as follows. For any 3 digit integer (written xyz), . If c and k are 3 digit integers, and , what is the value of c - k?

Choices

A B C D E Cannot be determined.
Since a, b, c, d, and e are consecutive odd integers, their mean, or average, will be the middle number in the sequence: c. So the question is what information is sufficient to determine the value of c. (1) This means that two adjacent odd numbers, and then 2 odd numbers later, are all prime. This happens an infinite number of times, but more to the point, it happens more than once. For example, 11, 13, and 17 are all prime, as are 41, 43, and 47. NOT SUFFICIENT (2) Since a and d are odd integers separated by each other by 2 other odd integers (b and c), that means that d is 6 more than a (i.e., 3 odd integers more). So a + (6+a) = 24 2a = 18

a = 9, and c will be 13, our arithmetic mean. SUFFICIENT The credited response is B: statement (2) alone is sufficient to answer the question, but statement (1) alone is not. In the infinite sequence of odd integers, let a, b, c, d, and e be five consecutive members of the sequence. What is the arithmetic mean of these 5 numbers? (1) b, c, and e are all prime numbers. (2) a + d = 24

Choices

A Statement (1) ALONE is sufficient, but statement (2) alone is not sufficient. B Statement (2) ALONE is sufficient, but statement (1) alone is not sufficient. C BOTH statements TOGETHER are sufficient, but NEITHER statement ALONE is sufficient. D EACH statement ALONE is sufficient. E Statements (1) and (2) TOGETHER are NOT sufficient to answer the question asked, and additional data are needed.
Expanding the terms, the question is asking is

Noting that the numerators and denominators of the two fractions are just the flip of each other, this is really asking is

, where Now, generally speaking, when is n greater than (i) If (ii) If (iii) If (iv) If , then , for instance n = 2, 2 > 1/2 , then , then , then ?

In other words, for our question, we need to know both which is greater, 2a - b or 2b - a, and we also need to know if one or both of them is negative. (1) Is 2a - b > 2b - a? This can be restated as, Is 3a > 3b? Or, Is a>b? If a<b, as in Statement (1), then 2a-b < 2b-a. In other words, , but we don't know if it's positive or negative, we don't know if it fits into category ii, iii, or iv. If, for example, a = 2 and b = 3, then . But if, for example, a = 1 and b = 3, then . NOT SUFFICIENT (2) Given that 2a < b, we now know that 2a - b <0. This alone is still insufficient, however, as an example can show: let a = -2 and b = -3, then and . But if a = -3 and b = -2, then the reverse is true. NOT SUFFICIENT. But together are they sufficient? Still not sufficient. Since if a = -3 and b = -1, then and , so . and , so

and

, so

But if a = -1 and b = 3, then

and

, so

Answer choice E is the credited response - both answer choices together are not sufficient.

If (1) (2)

, where

, then is

Choices

A Statement (1) ALONE is sufficient, but statement (2) alone is not sufficient. B Statement (2) ALONE is sufficient, but statement (1) alone is not sufficient. C BOTH statements TOGETHER are sufficient, but NEITHER statement ALONE is sufficient. D EACH statement ALONE is sufficient. E Statements (1) and (2) TOGETHER are NOT sufficient to answer the question asked, and additional data are needed. Your answer was correct!

You might also like